Matemáticas, pregunta formulada por btsjpymjnnj, hace 8 meses

pongo coronita por favorrrrrr
3\left(t^3-4t^2+6t-3\right)+5\left(-t^2+2t^2-9t+11\right)

Respuestas a la pregunta

Contestado por daniscole
1

Respuesta:

solo simplificas hasta que ya no se pueda

Adjuntos:
Otras preguntas